LSAT and Law School Admissions Forum

Get expert LSAT preparation and law school admissions advice from PowerScore Test Preparation.

 actionjackson
  • Posts: 22
  • Joined: Nov 22, 2016
|
#32246
For this question, as I was running out of time for the section, I was between answer choices B and D. Initially, I had selected D but then switched to answer choice B. I categorized this question type as: flaw in the reasoning. During my analysis of the stimulus I had determined that causal reasoning was present. With T= testosterone and E= estrogen I had quickly diagrammed:

T :arrow: ^HD (promote heart disease)

E :arrow: vHD (inhibit heart disease)

Hopefully so far so good. I then proceeded to the answer choices armed with the knowledge of the ways we deal with causality on this test. I think where I went wrong was I looked at answer choice D (after selecting it), looked at answer choice B (after having selected D) and said to myself "D provides that what is discussed in the stimulus is correlated, but (I had thought) there was no indication of a correlation in the stimulus" (there totally is: Age goes up, hormones adjust according to one's sex, fluctuation of hormones with the more heart disease, three things that are seemingly related). Whereas B seemed to provide one of the assumptions inherent in any argument involving causality on this exam. Going over the question and answer choices I think I now know why B is wrong and D correct but again want to confirm that my thinking is accurate. B is wrong because of that word "only" when the stimulus concluded that the hormones tend to inhibit heart disease? D is correct because it accurately summarizes the flaw present in the argument?
 Emily Haney-Caron
PowerScore Staff
  • PowerScore Staff
  • Posts: 577
  • Joined: Jan 12, 2012
|
#32274
Hi actionjackson,

Your analysis for why D is correct is spot-on. Great job! The more you can work it through for yourself, the quicker you'll master this test, so I'm always excited to see a student grappling with things and explaining where you think you went wrong. :-D

For B, I'm not sure I completely follow what you're saying, so let's just talk it through. B could be 100% false, and it wouldn't hurt the argument at all - keep in mind that this is an assumption question and you've always got the assumption negation technique as a tool to use! If we negated B (they are NOT the only hormones that promote or inhibit) would that weaken the argument? Nope. Why not? Well, because even if other hormones also play the same function, that doesn't make it so that estrogen and testosterone don't. There can be multiple causes, so showing another cause doesn't take away from the one we're talking about here.

Does that make sense?
 actionjackson
  • Posts: 22
  • Joined: Nov 22, 2016
|
#32319
Emily Haney-Caron wrote:Hi actionjackson,

Your analysis for why D is correct is spot-on. Great job! The more you can work it through for yourself, the quicker you'll master this test, so I'm always excited to see a student grappling with things and explaining where you think you went wrong. :-D

For B, I'm not sure I completely follow what you're saying, so let's just talk it through. B could be 100% false, and it wouldn't hurt the argument at all - keep in mind that this is an assumption question and you've always got the assumption negation technique as a tool to use! If we negated B (they are NOT the only hormones that promote or inhibit) would that weaken the argument? Nope. Why not? Well, because even if other hormones also play the same function, that doesn't make it so that estrogen and testosterone don't. There can be multiple causes, so showing another cause doesn't take away from the one we're talking about here.

Does that make sense?
Thank you Emily. I categorized the question type as one of a flaw in the reasoning. That might have been where I went wrong.I didn't properly categorize this as an assumption question. As a quick follow up, would this be considered a supported assumption as opposed to a defender assumption?
 Emily Haney-Caron
PowerScore Staff
  • PowerScore Staff
  • Posts: 577
  • Joined: Jan 12, 2012
|
#32327
Hi actionjackson,

Yep, I'd consider this one a supporter. There's a really great forum post on distinguishing between the two which you might also find helpful, here: lsat/viewtopic.php?f=12&t=7606 Sounds like you're on the right track, though!
 EL16
  • Posts: 45
  • Joined: Jul 06, 2017
|
#38029
Hi,

I got this question and understood it correctly, but was looking at the discussion here just for some further support. In the previous discussion here, this question stem is being categorized as an assumption. However, I thought it was a flaw in the reasoning question with a mistaken cause/effect relationship? Am I incorrect and this is actually an assumption?

Thanks!
Elana
 EL16
  • Posts: 45
  • Joined: Jul 06, 2017
|
#38182
Hello,

I just wanted to follow up with my previous question and make sure it didn't get skipped over!

Thanks,
Elana
 Adam Tyson
PowerScore Staff
  • PowerScore Staff
  • Posts: 5153
  • Joined: Apr 14, 2011
|
#38340
It's a little of both, EL16! The stem tells us clearly that the argument is flawed and that we want to know why, so that's a Flaw in the Reasoning stem. But it also tells us it is flawed because it made a bad assumption, and they want us to identify that assumption, so it's an assumption question. Since the author already told us what was wrong - a bad assumption - we don't need to describe the flaw, which is the task in an ordinary Flaw question. Instead, we are tasked with just finding that bad assumption. So, attack it like an ordinary assumption question, looking for a supporter or a defender and testing contenders (if you have more than one) with the Negation Technique.

I often tell my students that ALL flaws are, essentially, based on bad assumptions, but we usually want a more detailed answer than just " the author made a bad assumption". For example, a source argument is an argument based on the assumption that a bad person cannot make a good argument. A causal argument is flawed because it assumes no alternate cause is possible, that they aren't reversed, that the data is good, etc. A survey question might be flawed because the author assumed that the sample was representative, or that the questions were unbiased, or that the answers were truthful and accurate. The list goes on and on like that. Flaws and assumptions are inextricably intertwined, and many flaw prephrases should begin with "the author assumed without justification that..." (and many answer choices do just that).

Thanks for the question, giving us a chance to clarify this oddball question!
 MikeJones
  • Posts: 31
  • Joined: Oct 02, 2017
|
#41090
Hi. I've been having some trouble isolating certain elements in correlation causation arguments. I just want to make sure I'm seeing this one right since there seems to be more than one correlation relationship in this stimulus.

The first one obviously occurs in the first sentence: risk of heart disease is correlated with age in both men and women.

The other two appear to be:

1: More testosterone, higher chance of heart disease in men.
2. Less estrogen, less of a chance of heart disease in women.

The conclusion makes causal statement based on the last two correlations. I guess my question is how does the first correlation tie into this argument? I don't see any causal statements in regard to age, though it is obviously a factor in the hormones present in both sexes. Is my thinking correct here?
 James Finch
PowerScore Staff
  • PowerScore Staff
  • Posts: 943
  • Joined: Sep 06, 2017
|
#41665
Hi Mike,

The first sentence doesn't serve as a causal argument, but as a premise for the causal arguments made in the stimulus as a whole. We are meant to simply accept it as fact, and combine the information that it gives us about age and heart disease in men and women with the information given in the second sentence about the hormone levels in men and women at different ages to then draw a correlation between hormone levels in men and women as they age and risk of heart disease. Then the conclusion makes a causal argument, when all that has been presented up to that point is correlative evidence. Here's a diagram of the logic in the stimulus:

Young men more likely than young women to get heart disease + Young men have high testosterone :arrow: Testosterone promotes heart disease;

and,

Postmenopausal women more likely to get heart disease + After menopause women lose estrogen :arrow: Estrogen inhibits heart disease

This is questionable logic because correlation does not automatically mean causation.

Hope this clears things up!
User avatar
 sunshine123
  • Posts: 44
  • Joined: Jul 18, 2022
|
#96338
Hello,

The official Powerscore Logical Reasoning Bible reads that "when an LSAT speaker concludes that one occurrence definitively caused another, that speaker also assumes that the stated cause is the ONLY possible cause of the effect and that consequently the stated cause will always produce the effect"(p.291). Assuming the present stimulus makes such a definitive causal statement, then we can conclude that the author does assume that estrogen and testosterone are the only hormones that promote or inhibit heart disease, or that B is the credited response.

However, since it is not, I wonder if that's because the author is not really making any such definitive causal claims: the author merely claims that testosterone and estrogen TEND to PROMOTE heart disease, thus allowing room for other possible causes. Does all that sound plausible at all??

Thank you,
Sunshine

Get the most out of your LSAT Prep Plus subscription.

Analyze and track your performance with our Testing and Analytics Package.